Katzung Pharmacology Review 10e 100 Q Exam#2

Réussis tes devoirs et examens dès maintenant avec Quizwiz!

Question 12: A 29-year-old accountant has recurrent episodes of tachycardia that sometimes convert to sinus rhythm spontaneously but more often require medical treatment. A drug that is commonly given as an intravenous bolus for the purpose of converting AV nodal tachycardias to normal sinus rhythm is:

• adenosine The correct answer is A. Explanation: Adenosine is favored for the prompt conversion of atrioventricular nodal rhythms to normal sinus rhythm. Adenosine has a very short duration of action (seconds) and good efficacy.

Question 28: A 56-year-old woman has arthritis of the knees that limits her activity and post-herpetic neuralgia on her torso after an episode of herpes zoster. Which of the following drugs is used topically to control arthritic pain and post-herpetic neuralgia?

• capsaicin The correct answer is C. Explanation: Substance P is the endogenous peptide closely associated with peripheral pain transmission, and capsaicin (the "hot" component from hot peppers) is an antagonist.

Question 5: The mechanism of local anesthetic action of cocaine is

• cocaine The correct answer is E. Explanation: Local anesthetics block voltage-dependent sodium channels in excitable tissues including nerves, decreasing action-potential conduction. Rapidly firing fibers are more sensitive than slowly firing nerve fibers. Cocaine has this action and also blocks the reuptake of norepinephrine at sympathetic neuroeffector junctions with effects on both the heart and the CNS.

Question 77: Lidocaine is commonly used as a local anesthetic. If a patient mistakenly receives a toxic dose of lidocaine intravenously, the patient is likely to exhibit

• lidocaine The correct answer is E. Seizures and coma Explanation: Intravenous lidocaine causes typical local anesthetic toxicity including central nervous system stimulation with possible seizures. Cardiovascular depression may occur, but it is usually minor.

Question 51: Naloxone will not antagonize or reverse

• naloxone The correct answer is E. Respiratory depression caused by overdose of nefazodone Explanation: Naloxone is an opioid μ-receptor antagonist and will oppose the actions of opioids at this class of receptors including analgesia, miosis, and symptoms of opioid overdose including respiratory depression. However, respiratory depression due to nefazodone is not exerted via the μ-opioid receptor.

Question 61: A 52-year-old plumber comes to the office with a complaint of periodic onset of chest pain, described as a sensation of heavy pressure over the sternum that comes on when he exercises and disappears within 15 min when he stops. After a full physical examination and further evaluation, you make the diagnosis of angina of effort. A drug that is useful in angina but causes constipation, edema, and increased cardiac size is

• verapamil The correct answer is E. Explanation: Verapamil (and diltiazem) is useful for prophylaxis of both effort and vasospastic angina. Calcium blockers reduce cardiac work and oxygen demand. They also cause constipation and sometimes peripheral edema that is not associated with heart failure.

Question 86: Which of the following, an inhibitor of fungal cell membrane synthesis, is effective intravenously in the management of disseminated infections due to Aspergillus or Candida species?

• voriconazole The correct answer is E. Explanation: Amphotericin B and caspofungin are active against many systemic fungal infections, but they interfere with fungal cell wall functions. Voriconazole, an azole antifungal, like fluconazole and itraconazole, interferes with cell membrane permeability by inhibiting ergosterol synthesis.

Question 7: Hypercoagulability and dermal vascular necrosis resulting from protein C deficiency is known to be an early-appearing adverse effect of treatment with which of the following drugs?

• warfarin The correct answer is E. Explanation: Warfarin interferes with gamma carboxylation of clotting factors IX, X, VII, and II, and the anticlotting factors protein C and protein S. Alteplase carries the risk of bleeding and not thrombosis. Aspirin can cause gastrointestinal ulcers and bleeding. Heparin is the only drug on this list that can also cause thrombosis, but in the case of heparin, it is mediated not through protein C but rather through an immunologic reaction against heparin-platelet complexes.

Question 78: Which of the following is an oral antidiabetic drug that inhibits an enzyme in the gastrointestinal tract that converts polysaccharides to monosaccharides and thereby reduces postprandial hyperglycemia?

• acarbose The correct answer is A. Explanation: Acarbose inhibits polysaccharide breakdown. Glipizide is a secretagogue. Metformin acts in the liver. Rosiglitazone is a PPAR-γ activator, and sitagliptin is a DPP-IV inhibitor.

Question 57: Persons who ingest three or more alcoholic drinks daily can develop severe hepatotoxicity after doses of acetaminophen that are not toxic to individuals with normal liver function. This increased sensitivity to acetaminophen's toxicity is due to which of the following mechanisms?

• acetaminophen • poisoning by acetaminophen The correct answer is D. Increased activity of cytochrome P450 mixed function oxidase isozymes Explanation: Chronic alcohol use induces hepatic cytochrome (CYP) P450 mixed oxidase isozymes enzymes. Elevated CYP activity converts more acetaminophen into a toxic intermediate, which requires inactivation by glutathione (GSH). This alcohol-induced increase in CYP increases the risk of reaching a level of toxic acetaminophen metabolite that can overwhelm the liver's detoxification capacity and result in severe hepatotoxicity.

Question 42: Resistance to acyclovir is most commonly due to mutations in a viral gene that encodes a protein that

• acyclovir The correct answer is B. PhosphoArylates acyclovir Explanation: Many resistant strains of HSV commonly lack thymidine kinase, the enzyme involved in the viral-specific activation of acyclovir.

Question 6: Which of the following statements about acyclovir is accurate?

• acyclovir The correct answer is C. Bioactivated by viral thymidine kinase Explanation: Acyclovir is a guanosine analog activated by viral thymidine kinases of HSV and VZV to form acyclovir triphosphate, a competitive substrate for DNA polymerase resulting in chain termination when incorporated into viral DNA.

Question 25: A 16-year-old student has had asthma for 8 yr. The number of episodes of severe bronchospasm has increased recently, and you have been asked to review the therapeutic plan. Which of the following agents is most likely to be of immediate therapeutic value in relieving an acute bronchospastic attack?

• albuterol The correct answer is A. Explanation: Albuterol, metaproterenol, and terbutaline are rapid-onset, selective β2 agonists used as first-line therapy for acute asthma.

Question 59: A 67-year-old man with osteoporosis was being treated with once-weekly alendronate. This medication has the potential to cause which of the following unusual adverse effects?

• alendronate The correct answer is B. Esophageal irritation Explanation: The bisphosphonates can cause esophageal irritation, which is managed by drinking lots of fluids and staying upright after taking the drug. Blue skin is an adverse effect of amiodarone. Color vision can be affected by anti-TB treatment. Priapism can be caused by sildenafil in combination with nitrates, and tendinitis is a side effect for fluoroquinolones.

Question 91: The antibacterial action of aminoglycosides is due to their ability to

• aminoglycosides The correct answer is B. Bind to the 30S ribosomal subunit and block initiation of bacterial protein synthesis Explanation: B (45) The aminoglycoside antibiotics are bactericidal inhibitors of protein synthesis. You may recall that their actions continue well beyond their short half-lives because they exert a "postantibiotic action."

Question 32: A young man comes to a community clinic with a urogenital infection that, based on the Gram stain, appears to be due to Neisseria gonorrhoeae. Questioning suggests that the patient acquired the infection while vacationing abroad. The physician is concerned about drug resistance of the gonococcus. Which drug is least likely to be effective in the treatment of gonorrhea in this patient?

• amoxicillin The correct answer is A. Explanation: The drugs of choice for treatment of gonorrhea currently are the cephalosporins ceftriaxone and cefixime. Azithromycin and spectinomycin are alternatives. Resistance to amoxicillin is common.

Question 31: A 54-year-old woman was found to have node-positive breast cancer. Following her surgery, she was treated with a drug that prevents the conversion of testosterone to estradiol. The drug used for her treatment most likely was which of the following?

• anastrozole The correct answer is A. Explanation: Anastrozole prevents conversion of testosterone to estradiol. Ethinyl estradiol is an orally available form of estradiol. Finasteride is a 5-α-reductase inhibitor.Spironolactone is an androgen receptor antagonist (also known as K-sparing diuretic), and tamoxifen is a selective estrogen receptor modulator.

Question 33: Which of the following toxic compounds is correctly paired with an antidote that is used in the treatment of a patient poisoned with the toxic compound?

• antidotes The correct answer is D. Cyanide: hydroxocobalamin Explanation: In cyanide poisoning, the vitamin B12 analog hydroxocobalamin reacts with cyanide to form a nontoxic metabolite that is eliminated by the kidney.

Question 70: A 70-year-old woman with mild to moderate hypertension fell 2 yr ago during a spell of dizziness and broke her hip. During the last 18 mo, her blood pressure has increased. Now she is to be treated for a blood pressure of 170/100 mm Hg. When treating hypertension chronically, orthostatic hypotension is greatest with

• antihypertensive agents The correct answer is D. Peripherally acting α1 antagonists Explanation: ACE inhibitors, arteriolar dilators, and β blockers do not ordinarily cause orthostatic hypotension; venodilators do. Peripheral α1 antagonists block sympathetic effects on both arterioles and veins and thus may cause orthostatic hypotension, especially with the first few doses.

Question 24: Ms. B, a 40-year-old woman, has been receiving a slow IV infusion of phenylephrine. She is currently showing an excessive heart rate effect from this drug. A drug that blocks the heart rate effect of a slow IV infusion of phenylephrine is

• atropine The correct answer is A. Explanation: The increase in blood pressure caused by pressor drugs such as phenylephrine evokes a reflex slowing of heart rate that is mediated by the vagus nerve acting on muscarinic receptors in the sinoatrial node. This slowing can be blocked by antimuscarinic agents such as atropine.

Question 19: A patient admitted to the emergency department is vomiting blood. Her supine blood pressure is 100/60 mm Hg; sitting up, her BP is 50/0. Which of the following most accurately describes the probable autonomic response to the bleeding?

• autonomic nervous system The correct answer is B Rapid heart rate, dilated pupils, damp skin Explanation: Questions about the baroreceptor reflex are common; Figure 6-4 is very high yield. The major responses to hypotension are sympathetic discharge (choice B) and activation of the renin-angiotensin-aldosterone system. FIGURE 6-4 View Full Size | Save Figure | Download Slide (.ppt) Autonomic and hormonal control of cardiovascular function. Note that 2 feedback loops are present: the autonomic nervous system loop and the hormonal loop. Each major loop has several components. In the neuronal loop, sensory input to the vasomotor center is via afferent fibers in the ninth and tenth cranial (PANS) nerves. On the efferent side, the sympathetic nervous system directly influences 4 major variables: peripheral vascular resistance, heart rate, contractile force, and venous tone. The parasympathetic nervous system directly influences heart rate. In addition, angiotensin II directly increases peripheral vascular resistance (not shown), and sympathetic nervous system discharge directly increases renin secretion (not shown). Because these control mechanisms have evolved to maintain normal blood pressure, the net feedback effect of each loop is negative; feedback tends to compensate for the change in arterial blood pressure that evoked the response. Thus, decreased blood pressure due to blood loss would be compensated by increased sympathetic outflow and renin release. Conversely, elevated pressure due to the administration of a vasoconstrictor drug would cause reduced sympathetic outflow, decreased renin release, and increased parasympathetic (vagal) outflow. (Modified and reproduced, with permission, from Katzung BG, editor: Basic & Clinical Pharmacology, 12th ed. McGraw-Hill, 2012: Fig. 6-7.)

Question 75: A 64-year-old recipient of a kidney transplant was being treated with immunosuppressants. After several episodes of gout, the decision was made to treat his gout with the xanthine oxidase inhibitor allopurinol. The dose of which of the following of his immunosuppressant drugs should be reduced to avoid excessive bone marrow suppression due to a drug-drug interaction?

• azathioprine The correct answer is A. Explanation: Allopurinol interferes with the metabolism of azathioprine, increasing plasma levels of 6-mercaptopurine, which may result in potentially fatal blood dyscrasias. Concomitant use requires the dose of azathioprine to be reduced to 25%.

Question 79: A young man comes to a community clinic with a urogenital infection that, based on the Gram stain, appears to be due to Neisseria gonorrhoeae. Questioning suggests that the patient acquired the infection while vacationing abroad. The physician is concerned about drug resistance of the gonococcus. The physician is also concerned about the possibility of a nongonococcal urethritis in this patient. Several antibiotics in the list below are active in, nongonococcal urethritis, However, these infections, including those caused by C trachomatis, can usually be eradicated by the administration of a single dose of

• azithromycin The correct answer is A. Explanation: The only drug likely to be effective in nongonococcal urethritis in a single dose is azithromycin, which has an elimination half-life of several days. Other drugs used in nongonococcal urethritis include clindamycin, ofloxacin, and the tetracyclines.

Question 97: A patient with diffuse non-Hodgkin's lymphoma is treated with a combination drug regimen that includes bleomycin, cyclophosphamide, vincristine, doxorubicin, and prednisone. The patient's cumulative dose of bleomycin will be carefully monitored because high cumulative doses are associated with which of the following?

• bleomycin The correct answer is E. Pulmonary fibrosis Explanation: Bleomycin is one of the 4 drugs for which myelosuppression is not dose-limiting. The 3 others are cisplatin for nephrotoxicity, doxorubicin for cardiotoxicity, and vincristine for peripheral neuropathy.

Question 27: In a patient with familial combined hyperlipidemia that is associated with increased VLDL and LDL, which of the following drugs is most likely to increase plasma triglycerides while also decreasing plasma LDL?

• cholestyramine The correct answer is A. Explanation: Resins increase triglycerides (TGs), particularly in patients who are genetically predisposed to high TGs. Ezetimibe has no effect on TGs, but fibrates and niacin both lower TGs and lovastatin mostly lowers LDL cholesterol.

Question 72: Which of the following drugs is approved for primary prophylaxis in AIDS patients with low CD4 counts against infections due to Mycobacterium avium-intracellulare?

• clarithromycin The correct answer is C. Explanation: Azithromycin (not listed) or clarithromycin with or without rifabutin is recommended for primary prophylaxis against Mycobacterium avium complex (MAC) in patients with AIDS.

Question 49: Chronic heart failure is commonly treated with a combination of drugs that both improve symptoms and provide long-term benefits. Three drugs or drug groups that have been shown in clinical trials to provide benefits in patients with chronic heart failure are

• clinical trials The correct answer is A. ACE inhibitors, carvedilol, and spironolactone Explanation: While therapeutic strategies in chronic heart failure include use of several different drug classes including diuretics and positive inotropics, only 3 drugs or drug groups have been shown to provide benefits in terms of survival: ACE inhibitors, certain β blockers including carvedilol, and the aldosterone antagonists spironolactone and eplerenone.

Question 67: A new drug was administered to a group of normal volunteers in a phase 1 clinical trial. Intravenous bolus doses produced the changes in blood pressure and heart rate shown in the graph below. View Full Size | Save Figure | Download Slide (.ppt) The most probable receptor affinities of this new drug are

• clinical trials The correct answer is C. β1 and β2 only Explanation: The graph shows a marked decrease in diastolic BP and heart rate, with only a small, transient increase in systolic BP. These effects are characteristic of isoproterenoland similar β1, β2 agonists.

Question 34: A new drug for the prophylaxis of asthma is under development in a pharmaceutical company. Before human trials are begun, FDA regulations require that

• clinical trials The correct answer is E. The effect of the drug on reproduction be studied in at least 2 animal species Explanation: Before clinical trials can be carried out with a new drug, reproductive toxicity data must be provided for at least 2 species.

Question 65: Which statement is accurate?

• clonazepam The correct answer is B. Clonazepam is a useful antiseizure drug Explanation: Clonazepam, a benzodiazepine, is effective in the management of absence seizures and is also used in the treatment of bipolar disorder.

Question 14: A 40-year-old woman was being treated for chronic moderate hypertension. When she went on vacation and forgot her pills, her blood pressure rose markedly and she was admitted to the emergency service with blurred vision, severe headache, and retinal hemorrhages. A drug that is most likely to be followed by rebound hypertension if stopped suddenly is

• clonidine The correct answer is B. Explanation: Of the drugs listed, only clonidine, an α2 agonist, is associated with severe rebound hypertension if stopped suddenly. It is speculated that this effect is due to down-regulation of α2 receptors.

Question 22: A 73-year-old patient has chronic pulmonary dysfunction requiring daily hospital visits for respiratory therapy. She is hospitalized with pneumonia, and it is not clear whether the infection is community or hospital acquired. If she has a community-acquired pneumonia, coverage must be provided for pneumococci and atypical pathogens. In such a case, the most appropriate drug treatment in this patient is

• community acquired pneumonia The correct answer is B. Ceftriaxone plus erythromycin Explanation: In a community-acquired pneumonia, the wider spectrum cephalosporin ceftriaxone would cover typical organisms, and erythromycin would be active against the atypical organisms.

Question 54: A 31-year-old premenopausal woman has been using a combined oral contraceptive for 10 yr. As a result of this contraceptive use, she has a reduced risk of which of the following?

• contraceptives, oral The correct answer is D. Ovarian cancer Explanation: Combination hormonal contraceptives have clinical uses and beneficial effects in treatment of acne, hirsutism, and dysmenorrhea. In addition, with long-term use, they have been shown to reduce the risk of ovarian and endometrial cancer.

Question 37: In your pursuit of a Nobel Prize, you have been studying several intracellular enzymes. You recall that cyclooxygenase-1 and -2 are responsible for the

• cyclooxygenase 2 • cyclooxygenase 1 The correct answer is E. Synthesis of prostaglandins from arachidonate Explanation: The cyclooxygenase enzymes are responsible for cyclizing arachidonate to prostaglandin precursors.

Question 81: This cell cycle-nonspecific agent is commonly used as a component of cancer chemotherapy regimens, including those for non-Hodgkin's lymphoma and for breast cancers; administration of mercaptoethanesulfonate (mesna) decreases the risk of hematuria.

• cyclophosphamide The correct answer is A. Explanation: Cyclophosphamide is converted into acrolein, which damages the kidney. mesna can bind to and detoxify acrolein. Note that the dose-limiting toxicity ofcyclophosphamide is bone marrow suppression.

Question 47: Early in an anesthesia procedure, which includes the use of succinylcholine and halothane, a surgical patient develops severe muscle rigidity, hypertension, and hyperthermia. Management of this patient will almost certainly include the administration of

• dantrolene sodium The correct answer is B. Explanation: Malignant hyperthermia is a rare disorder characterized by massive calcium release within skeletal muscle triggered by use of succinylcholine in anesthesia protocols.Dantrolene is given to block calcium release.

Question 4: A 42-year-old woman developed a syndrome of polyuria, thirst, and hypernatremia after surgical removal of part of her pituitary gland. These signs and symptoms will be treated with which of the following?

• deamino arginine vasopressin The correct answer is B. Desmopressin Explanation: This woman's central diabetes insipidus is due to insufficient posterior pituitary production of vasopressin. Desmopressin, a selective vasopressin V2 receptor agonist, can be administered orally, nasally, or parenterally to treat central diabetes insipidus..

Question 60: A 2-year-old girl was brought to the emergency department because of vomiting, bloody diarrhea, and hypotension. An abdominal x-ray film showed multiple radiopaque pills, and a relative at the child's home reported the discovery of an open bottle of iron pills behind a large piece of furniture. In addition to supportive care, treatment of the child is most likely to include which of the following actions?

• deferoxamine The correct answer is B. Intravenous administration of deferoxamine Explanation: Deferoxamine, a chelator with high selectivity and affinity for iron, is used intravenously for acute iron poisoning. Activated charcoal does not bind iron. Acetylcysteine is used for acetaminophen poisoning. Pralidoxime is used for organophosphate poisoning. Intravenous EDTA is used for lead poisoning; oral EDTA does not significantly reduce iron absorption after an oral iron overdose.

Question 3: A 35-year-old patient is brought to the emergency department in a drug-induced coma. Blood samples taken over the next several hours show a declining drug concentration, as shown in the graph below. Which of the following drugs is the most likely cause of this patient's coma? View Full Size | Save Figure | Download Slide (.ppt)

• diazepam The correct answer is B. Explanation: The graph shows first-order elimination of the drug in question. Aspirin, ethanol, and phenytoin are eliminated mainly by zero-order kinetics. Diazepam is the only drug in the list that is eliminated by first-order kinetics.

Question 62: You are on the Hospital Pharmacy Committee and revising the formulary. Relative to loratadine, diphenhydramine is more likely to

• diphenhydramine The correct answer is D. Have efficacy in the prevention of motion sickness Explanation: Diphenhydramine is a first-generation anti-H1 blocker with significant anti-motion sickness and sedative actions. Loratadine is a second-generation antihistamine with neither of these effects. Neither drug is effective in asthma or GERD, and neither blocks drug-metabolizing enzymes.

Question 35: A patient with diffuse non-Hodgkin's lymphoma is treated with a combination drug regimen that includes bleomycin, cyclophosphamide, vincristine, doxorubicin, and prednisone. Dexrazoxane is thought to protect against the distinctive toxicity of which drug in this patient's regimen?

• doxorubicin The correct answer is C. Explanation: The toxicity of doxorubicin (one of the 4 drugs with unique dose-limiting toxicity; see information below) can be mitigated by dexrazoxane. (A patient with diffuse non-Hodgkin's lymphoma is treated with a combination drug regimen that includes bleomycin, cyclophosphamide, vincristine, doxorubicin, andprednisone. The patient's cumulative dose of bleomycin will be carefully monitored because high cumulative doses are associated with pulmonary fibrosis.)

Question 52: A 26-year-old woman with chronic bronchitis lives in a region of the country where winter conditions are harsh. Her physician recommends prophylactic use of oral doxycycline, to be taken once daily, during the winter season. Which statement about the characteristics and use of doxycycline in this patient is accurate?

• doxycycline The correct answer is D. The patient should discontinue the tetracycline if she becomes pregnant. Explanation: Tetracycline use during pregnancy is discouraged since fetal exposure to these antibiotics may ultimately lead to irregularities in bone growth and dentition. In addition, gastrointestinal effects and the potential for hepatic dysfunction are increased in the pregnant patient.

Question 93: An example of a phase I drug-metabolizing reaction is

• drug metabolism The correct answer is C. Hydroxylation Explanation: Acetylation, glucuronidation, methylation, and sulfation are phase 2 conjugation reactions.

Question 58: A patient with a myocardial infarction, heart failure, and an arrhythmia is to receive lidocaine by constant IV infusion. The target plasma concentration is 3 mg/L. The pharmacokinetic parameters for lidocaine in the general population are Vd 70 L, CL 35 L/h, and t1/2 1.4 h. An infusion is begun. The plasma concentration of lidocaine is measured 2.8 h later and reported to be 2.4 mg/L.

• drug steady state concentration The correct answer is C. 3.2 mg/L Explanation: During a continuous IV infusion, the plasma concentration approaches steady state according to the algorithm: 50% at 1 half-life, 75% at 2, etc. Since the sample was taken at two half-lives, the steady state concentration will be in the range of four-thirds the measured concentration (2.4/0.75) or 3.2 mg/L.

Question 8: A 15-year-old girl is admitted complaining of palpitations and shortness of breath. An ECG reveals sinus tachycardia with a heart rate of 160 bpm. A drug suitable for producing a brief (5- to 15-min) increase in cardiac vagal effects is..

• edrophonium The correct answer is B. Explanation: Symptomatic paroxysmal sinus tachycardia often occurs in young patients and can sometimes be converted to normal sinus rhythm with increased vagal discharge. Brief amplification (5-15 min) of the vagal effects on heart rate can be accomplished with a short-acting cholinesterase inhibitor such as edrophonium. Pyridostigmine (E) would have a much longer action (4-6 hr).

Question 20: A 30-year-old male patient who is HIV positive has a CD4 T lymphocyte count of 450 cells/μL (normal, 600-1500 cells/μL) and a viral RNA load of 11,000 copies/mL. His treatment involves a 3-drug antiviral regimen (HAART) consisting of zidovudine, didanosine, and efavirenz. Efavirenz limits HIV infection by Serving as an allosteric inhibitor of HIV reverse transcriptase

• efavirenz The correct answer is E. Explanation: Efavirenz is used in highly active antiretroviral therapy (HAART) regimens against HIV. The drug is an allosteric inhibitor of HIV reverse transcriptase and does not bind to the active site of the enzyme.

Question 64: A 34-year-old woman in her second trimester of pregnancy presented with a tender, red, swollen calf that was diagnosed as a deep vein thrombosis (DVT) and was treated successfully. However, because of the high risk of recurrence of the DVT, she was treated with an anticoagulant for the remainder of her pregnancy. The drug used most likely was which of the following?

• enoxaparin The correct answer is C. Explanation: Deep vein thromboses are located in the venous side of the circulation, making them less responsive to the antiplatelet agents (aspirin, clopidogrel). Warfarin is teratogenic and is contraindicated in pregnancy. Lepirudin is the thrombin inhibitor that can be used parenterally only. A low-molecular-weight (LMW) heparin is the drug of choice to treat and prevent deep vein thromboses and is safe to use during pregnancy.

Question 94: Several drugs used in patients with advanced Parkinson's disease allow patients to lower their dose of L-dopa/ carbidopa, and thus reduce the incidence of L-dopa-induced dyskinesias. These drugs also decrease the amount of "off" time for the patient. Which drug is used in this way but does not, if used alone, ameliorate the symptoms of early Parkinson's disease or enhance CNS dopaminergic activity?

• entacapone The correct answer is C. Explanation: Entacapone is a catechol-O-methyltransferase inhibitor that enhances the action of levodopa by preventing its metabolism in the blood and peripheral tissues. It does not cross the blood-brain barrier and if used alone will not ameliorate symptoms of Parkinson's disease.

Question 87: A 57-year-old woman with moderately severe rheumatoid arthritis was treated with etanercept. Which of the following proteins is the direct target of etanercept?

• etanercept The correct answer is E. Tumor necrosis factor-α (TNF-α) Explanation: Etanercept is a recombinant fusion protein consisting of 2 soluble TNF-α receptor moieties linked to human IgG. It binds to TNF-α and inhibits its action.

Question 56: Mental retardation, microcephaly, and underdevelopment of the midface region in an infant is associated with chronic heavy maternal use during pregnancy of which of the following?

• ethanol The correct answer is C. Explanation: Mental retardation, microcephaly, and facial dysmorphia are characteristics of fetal alcohol syndrome, caused by excessive use of ethanol during pregnancy.

Question 73: A young patient has a seizure disorder with recurrent contractions starting in muscles of the right hand that spread through the arm and the right side of the face. The attacks last for only a minute or two with no loss of consciousness. Which of the following drugs is least likely to be useful in the treatment of this patient?

• ethosuximide The correct answer is B. Explanation: Simple partial seizures can have the characteristics of the "jacksonian march." Many drugs are used in management including those listed, with the exception ofethosuximide, which is not effective in partial seizures or generalized tonic-clonic seizure states.

Question 90: A male patient with AIDS has a CD4 T lymphocyte count of 50 cells/μL (normal, 600-1500 cells/μL). He is being maintained on a multidrug regimen consisting of acyclovir, clarithromycin, dronabinol, fluconazole, lamivudine, indinavir, trimethoprim, sulfamethoxazole, and zidovudine. The drug that provides prophylaxis against cryptococcal infections of the meninges is

• fluconazole The correct answer is C. Explanation: Fluconazole is the only antifungal drug listed. It is the drug of choice for treatment and secondary prophylaxis against cryptococcal meningitis.

Question 30: An individual who ingested an antifreeze solution containing ethylene glycol was brought to a hospital emergency department. In an attempt to prevent severe acidosis and renal damage, the patient was given fomepizole. Fomepizole is useful in ethylene glycol poisoning because it inhibits which of the following?

• fomepizole The correct answer is A. Alcohol dehydrogenase Explanation: Fomepizole, an antidote for ethylene glycol and methanol poisoning, inhibits alcohol dehydrogenase, which converts ethylene glycol and methanol to toxic metabolites.

Question 43: A 55-year-old executive has cardiomyopathy and congestive heart failure. He is being treated with diuretics. The mechanism of action of furosemide is best described as

• furosemide The correct answer is B. Explanation: Furosemide acts on the ascending limb of the loop of Henle and inhibits the major transporter in this segment, a Na+/K+/2Cl- transporter.

Question 11: A 60-year-old man with a history of a mild myocardial infarction was discovered to have low serum HDL cholesterol and moderately high serum triglyceride level. His serum total and LDL cholesterol concentrations were well below the upper limit of normal. Which of the following drugs is likely to result in the greatest lowering of this patient's serum triglyceride concentration and elevation of his serum HDL cholesterol concentration?

• gemfibrozil The correct answer is C. Explanation: Fibrates are the main triglyceride-lowering drugs. Cholestyramine, ezetimibe, and lovastatin mainly lower LDL cholesterol, whereas rosiglitazone is an antidiabetic drug that increases insulin sensitivity through PPAR-γ activation.

Question 82: Which of the following drugs is most likely to cause hypoglycemia when used as monotherapy in the treatment of a patient with type 2 diabetes?

• glipizide The correct answer is B. Explanation: Glipizide is the only agent in this list that can stimulate insulin secretion, potentially leading to hypoglycemia.

Question 23: A 48-year-old surgical patient was anesthetized with an intravenous bolus dose of propofol, then maintained on isoflurane with vecuronium as the skeletal muscle relaxant. At the end of the surgical procedure, she was given pyridostigmine and glycopyrrolate. The rationale for use of glycopyrrolate was to

• glycopyrrolate The correct answer is C. Counter the potential cardiac effects of the acetylcholinesterase inhibitor D Explanation: The acetylcholinesterase inhibitor pyridostigmine can reverse skeletal muscle relaxation (caused by vecuronium) but may also cause bradycardia. The later effect can be prevented by use of glycopyrrolate, which has muscarinic receptor blocking action.

Question 66: A 57-year-old contractor with hypertension has been treated by 2 different physicians. He now comes to the emergency department with a severe reaction. Questioning reveals that he has been taking captopril and spironolactone. This combination is usually ill-advised because of the risk of

• hyperkalemia The correct answer is C. Explanation: Spironolactone inhibits potassium excretion in the kidney by blocking aldosterone. Captopril reduces angiotensin II levels and secondarily reduces aldosterone. The combination may increase serum potassium to dangerous levels.

Question 80: A 57-year-old man presented with signs and symptoms of acute gout that included intense pain in the first metatarsophalangeal joint of his right big toe of 1 day's duration and a joint that was swollen, tender, and red. Examination of synovial fluid removed from the joint revealed crystals of uric acid. The patient had a serum uric acid concentration of 10 mg/dL (normal 3.0-8.2 mg/dL). This was the patient's first episode of gout. He did not have any other medical illnesses and was not taking any medications. Which of the following is the most appropriate drug for immediate treatment of this acute attack of gout?

• indomethacin The correct answer is B. Explanation: Treatment of gout falls into 2 categories: 1) to treat the acute attack where the goal is to reduce pain and inflammation, and 2) to prevent attacks by reducing the uric acid pool through inhibition of uric acid buildup (allopurinol) or by enhanced elimination (probenecid). Indomethacin is an NSAID that can reduce pain and inflammation.Morphine will reduce only the pain but not the inflammation, and methotrexate is more effective for immuno-inflammatory disorders.

Question 38: The mechanism of high-level isoniazid (INH) resistance of M tuberculosis is

• isoniazid The correct answer is E. Reduced expression of the katG gene Explanation: Mutations in the katG gene result in the underproduction of mycobacterial catalase-peroxidase, an enzyme that bioactivates INH, facilitating its interaction with its "target" ketoacyl carrier protein sythetase. The result is high level resistance to INH. Mutations in the inhA gene result in low-level resistance with cross-resistance topyrazinamide.

Question 15: A 24-year-old man with a history of partial seizures has been treated with standard anticonvulsants for several years. He is currently taking valproic acid, which is not fully effective, and his neurologist prescribes another drug approved for adjunctive use in partial seizures. Unfortunately, the patient develops a toxic epidermal necrolysis. The second drug prescribed was

• lamotrigine The correct answer is D. Explanation: A number of antiseizure drugs have caused serious toxicities including hepatotoxicity with both valproic acid and felbamate. In the case of lamotrigine, which has been commonly used in the myoclonic seizures, toxic epidermal necrolysis (Stevens-Johnson syndrome) has occurred.

Question 68: A 65-year-old man has chronic open-angle glaucoma. The drug that is most likely to have therapeutic value for this condition is

• latanoprost The correct answer is C. Explanation: Mannitol is sometimes used to rapidly reduce intraocular pressure in acute angle-closure glaucoma. Of the drugs listed, only latanoprost is used in chronic glaucoma.

Question 36: Anticoagulation is needed immediately in a patient with deep vein thrombosis. The patient has a history of heparin-induced thrombocytopenia. Which of the following is the most appropriate drug for parenteral administration in this patient?

• lepirudin The correct answer is B. Explanation: Deep vein thromboses are located in the venous side of the circulation, making them less responsive to the antiplatelet agents (eptifibatide, clopidogrel). Unfractionatedheparin carries the risk of triggering more heparin-induced thrombocytopenia. Warfarin will take several days to achieve a therapeutic effect. Lepirudin (direct thrombininhibitor) is the drug of choice for fast anticoagulation in patients with heparin-induced thrombocytopenia.

Question 17: A 5-year-old boy was diagnosed as having an intestinal infection with Enterobius vermicularis (pinworm). He should be treated with?

• mebendazole The correct answer is B. Explanation: Mebendazole is the primary drug for treatment of pinworm, roundworm, and whipworm infections. The drug is contraindicated in pregnancy. Mebendazole and thiabendazole (a more toxic azole) are inhibitors of microtubule synthesis in nematodes.

Question 44: Long-term use of meperidine for analgesia is avoided because the accumulation of a metabolite, normeperidine, is associated with risk of

• meperidine The correct answer is E. Seizures Explanation: Meperidine is a strong opioid agonist with analgesic efficacy equivalent to that of morphine. The drug has a muscarinic blocking action and does not cause miosis or contraction of biliary smooth muscle. With long-term use, its metabolite normeperidine accumulates and may cause seizures.

Question 13: A 45-year-old woman suffers from abdominal pain and bloody diarrhea that has been diagnosed as Crohn's disease. Which of the following is a first-line drug for treatment of Crohn's disease that acts locally in the gastrointestinal tract to provide an anti-inflammatory effect?

• mesalamine The correct answer is D. Explanation: Mesalamine is a form of 5-aminosalicylic acid (5-ASA) used as first-line treatment for inflammatory bowel disease. Aluminum hydroxide is an antacid used for symptomatic relief of heartburn. Metoclopramide is a prokinetic agent. Misoprostol and ranitidine are used for acid-peptic disease.

Question 71: A patient is brought to the emergency department suffering from an overdose of an illicit drug. She is agitated, has disordered thought processes, suffers from paranoia, and "hears voices." Her physical symptoms include tachycardia, hyperreflexia, and hyperthermia. The drug most likely to be responsible for her condition is

• methamphetamine The correct answer is E. Explanation: The signs and symptoms are those of high-dose abuse of dextroamphetamine or methamphetamine. There is no specific antidote, and supportive measures are directed toward protection against cardiac arrhythmias and seizures and control of body temperature.

Question 84: Which of the following is a drug that inhibits the synthesis of thyroid hormone by preventing coupling of iodotyrosine molecules?

• methimazole The correct answer is D. Explanation: Methimazole [and propylthiouracil (PTU)] is a small sulfur-containing thioamide that inhibits thyroid hormone synthesis by blocking peroxidase catalyzed reactions, iodination of thyroglobulin, and coupling of DIT and MIT (see Figure 38-1). Dexamethasone is an orally active corticosteroid, lithium is a mood stabilizer, andpropranolol is a β-blocker capable of inhibiting the peripheral conversion of T4 to T3. FIGURE 38-1 View Full Size | Save Figure | Download Slide (.ppt) Sites of action of some antithyroid drugs. I−, iodide ion; I°, elemental iodine. Not shown: radioactive iodine (131I), which destroys the gland through radiation. (Reproduced, with permission, from Katzung BG, editor: Basic & Clinical Pharmacology, 12th ed. McGraw-Hill, 2012: Fig. 38-1.)

Question 21: A patient with a 30-yr history of type 1 diabetes comes to you with a complaint of bloating and sour belching after meals. On several occasions, vomiting has occurred after a meal. Evaluation reveals delayed emptying of the stomach, and you diagnose diabetic gastroparesis. Which drug would be most useful in this patient?

• metoclopramide The correct answer is B. Explanation: Metoclopramide, a dopamine D2 receptor antagonist, is a prokinetic drug that can be used to increase gastric emptying and intestinal motility in patients with diabetes-associated gastric paresis. Famotidine is a histamine H2 receptor antagonist used for acid-peptic disease. Misoprostol is a prostaglandin E1 analog used for acid-peptic disease and for medical abortions. Omeprazole is a proton pump inhibitor used for acid-peptic disease, and ondansetron is a serotonin 5-HT3 receptor antagonist used as an antiemetic.

Question 69: Although fentanyl or one of its congeners is usually administered in the early stages of a general anesthesia procedure, it is likely that the patient will receive an injection of morphine during the last phase. The rationale for switching to morphine is that the drug has

• morphine The correct answer is A. A longer duration of action Explanation: Fentanyl is much shorter acting than morphine. Both drugs are μ-receptor activators, equivalent in terms of analgesic activity and reversible by naloxone. Both drugs cause respiratory depression at high doses.

Question 83: A patient is admitted to the emergency department with signs and symptoms that could be due to either a muscarinic stimulant or an opioid. Which of the following is a common effect of both muscarinic stimulant drugs and opioids?

• muscarinic agonists • muscarinic antagonists The correct answer is E Miosis Explanation: Neither opioids nor muscarinic agonists decrease salivation, decrease sweating, or raise blood pressure. Opioids decrease peristalsis; muscarinics increase it. Both drug groups can cause miosis.

Question 76: A young patient with end-stage kidney disease receives a transplant from a living related donor who is HLA-identical and red blood cell ABO matched. To prevent rejection, the transplant recipient is treated with cyclosporine. Two years after initiating cyclosporine therapy, the patient developed evidence of cyclosporine-induced nephrotoxicity and hypertension. He was switched to an immunosuppressant that lacks renal toxicity and produces its immunosuppressant effect by inhibiting the de novo pathway of guanosine monophosphate (GMP) synthesis. The new immunosuppressant is which of the following drugs?

• mycophenolate mofetil The correct answer is C. Explanation: Mycophenolate mofetil is an immunosuppressant whose active metabolite inhibits de novo production of guanosine monophosphate (GMP). Lymphocytes are particularly sensitive to the antimetabolite effect of mycophenolate mofetil because they lack the alternative salvage pathway for GMP synthesis that is present in most cells.

Question 48: Which statement about nitric oxide is most correct?

• nitric oxide The correct answer is A. Nitric oxide is synthesized in vascular endothelium and the brain Explanation: Nitric oxide (NO) is not stored, it is synthesized on demand in response to acetylcholine and histamine in several tissues, including brain and endothelium. NO is released from the nitroprusside molecule.

Question 39: A 52-year-old plumber comes to the office with a complaint of periodic onset of chest pain, described as a sensation of heavy pressure over the sternum that comes on when he exercises and disappears within 15 min when he stops. After a full physical examination and further evaluation, you make the diagnosis of angina of effort. In considering medical therapy for this patient, which of the following best describes the beneficial action of nitroglycerin in this condition?

• nitroglycerin The correct answer is C. Dilation of systemic veins results in decreased diastolic cardiac size Explanation: Although nitrates may dilate large and medium coronary vessels, they have little effect on the arterioles in ischemic tissue, which are already dilated maximally by local ischemia. A major beneficial effect is venodilation, leading to reduction in cardiac size, which decreases diastolic fiber tension and reduces myocardial oxygen demand.

Question 55: A drug (Drug 1) was given as an IV bolus to a subject while blood pressure and heart rate were recorded as shown on the left side of the graph below. After recovery from the effects of Drug 1, a long-acting dose of Drug 2 was given. After the recorder was turned back on, Drug 1 was repeated with the results shown on the right side of the graph. Identify Drug 1 from the following list.

• norepinephrine The correct answer is G. Explanation: Drug 1 evokes a strong pressor effect and a bradycardia that is probably a reflex compensatory response. Thus, both norepinephrine and phenylephrine are possible answers. However, the effect of Drug 2 unmasks a tachycardia produced by Drug 1, so this agonist must be norepinephrine; phenylephrine does not have β agonist action.

Question 45: A 73-year-old patient has chronic pulmonary dysfunction requiring daily hospital visits for respiratory therapy. She is hospitalized with pneumonia, and it is not clear whether the infection is community or hospital acquired. If she has a hospital-acquired pneumonia, coverage must be provided for gram-negative bacteria (especially Pseudomonas aeruginosa) and for Staphylococcus aureus, many of which can be multiple drug-resistant organisms. In such a case, empiric treatment is likely to involve

• nosocomial pneumonia The correct answer is E. Vancomycin plus piperacillin/tazobactam Explanation: Single antimicrobial drug therapy would be inadequate coverage in a hospital-acquired pneumonia that could include possible infection due to multidrug-resistant staphylococci as well as gram-negative bacilli such as Pseudomonas aeruginosa. Vancomycin should cover gram-positive organisms, and piperacillin plus the penicillinase inhibitor would be active against most strains of likely gram-negative pathogens.

Question 53: A woman taking haloperidol developed a spectrum of adverse effects that included the amenorrhea-galactorrhea syndrome (Chiari-Frommel Syndrome) and extrapyramidal dysfunction. Another, newer, antipsychotic drug was prescribed which however caused weight gain and hyperglycemia due to a diabetogenic action. The drug prescribed was

• olanzapine The correct answer is E. Explanation: Significant weight gain and hyperglycemia due to a diabetogenic action occur with several atypical antipsychotics, especially clozapine (not listed) and olanzapine. Neurologic dysfunctions are less common with the atypical agents.

Question 95: After delivery of a healthy infant, a young woman begins to bleed extensively because her uterus has failed to contract. Which drug should be administered to this woman?

• oxytocin The correct answer is D. Oxytocin Explanation: The posterior pituitary hormone oxytocin contracts uterine smooth muscle. It is used to augment labor and, in this case, to treat postpartum uterine atony.

Question 63: The research division of a pharmaceutical corporation has characterized the receptor-blocking actions of 5 new drugs, each of which may have potential therapeutic value. The relative intensities of their blocking actions are shown in the following table. Because each of these drugs is lipophilic and can cross the blood-brain barrier, they are expected to have CNS effects. Based on the data shown in the table below, which drug is most likely to exacerbate the symptoms of Parkinson's disease? | Print Blocking Action on CNS Receptors Drug Adrenergic (Beta) Cholinergic (M) Dopaminergic (D2) GABAergic (A) A ++ +++ +++ None B None None None ++++ C None ++++ + None D + None +++ + E None + + +

• parkinson disease The correct answer is D. Drug D Explanation: D (21, 28) Antagonism at dopamine D2 receptors in the CNS is equated with parkinsonian symptoms (Drugs A and D). However, in the case of Drug A, this action would be offset by its ability to block muscarinic receptors. M blockers, like benztropine, improve tremor and rigidity in parkinsonism but have little effect on bradykinesia.

Question 41: After ingestion of a meal that included sardines, cheese, and red wine, a patient taking phenelzine experienced a hypertensive crisis. The most likely explanation for this untoward effect is that phenelzine

• phenelzine The correct answer is C. Inhibits the metabolism of catecholamines Explanation: Phenelzine, a rarely used antidepressant, is a potent MAO-B inhibitor and increases the amount of catecholamine transmitter stored in sympathetic nerve endings. When an indirectly acting sympathomimetic such as tyramine (found in some foods) avoids first-pass metabolism (due to MAO inhibition) and reaches the nerve endings, it can release large amounts of norepinephrine and cause a hypertensive crisis.

Question 50: Which one of the following pairs of drug and indication is accurate?

• pramipexol The correct answer is D. Pramipexole: Parkinson's disease Explanation: Pramipexole is a non-ergot dopamine agonist with high affinity for the D3 receptor. It is used as monotherapy in mild parkinsonism and together with levodopa in more advanced disease. Mental disturbances such as confusion, delusions, and impulsivity are more common with pramipexole than with levodopa.

Question 100: Identify Drug 2 from the following list. A drug (Drug 1) was given as an IV bolus to a subject while blood pressure and heart rate were recorded as shown on the left side of the graph below. After recovery from the effects of Drug 1, a long-acting dose of Drug 2 was given. After the recorder was turned back on, Drug 1 was repeated with the results shown on the right side of the graph.

• prazosin The correct answer is I. Explanation: See the information below. Drug 2 is an α blocker without β-blocking action. (Drug 1 evokes a strong pressor effect and a bradycardia that is probably a reflex compensatory response. Thus, both norepinephrine and phenylephrine are possible answers. However, the effect of Drug 2 unmasks a tachycardia produced by Drug 1, so this agonist must be norepinephrine; phenylephrine does not have β agonist action.)

Question 9: The long-term daily oral administration of therapeutic doses of prednisone results in which of the following?

• prednisone The correct answer is B. Explanation: Long-term use of prednisone has several side effects such as growth inhibition, diabetes, muscle wasting, and osteoporosis. Regarding the other options: Excess steroid use results in baldness. Corticosteroids (such as prednisone) can be used to treat some hemolytic anemias. Choice D is incorrect because it describes congenital adrenal hyperplasia, a condition in which the adrenal gland fails to make the hormones cortisol and aldosterone. Treatment would be to provide these hormones to restore physiological levels.

Question 2: Adding a progestin to the estrogenic component of hormone replacement therapy for postmenopausal women provides which of the following effects?

• progestins The correct answer is C. Reduces the risk of endometrial cancer Explanation: Progestin acts on the endometrium, where estrogen agonists can induce proliferation and endometrial cancer. Progestin has been shown to reduce this risk. Progestin toxicity includes increased blood pressure and a decrease in HDL. Long-term use of progestin can lead to a reduction in bone density.

Question 16: Propranolol and hydralazine have which of the following effects in common?

• propranolol • hydralazine The correct answer is C. Decreased mean arterial blood pressure Explanation: Hydralazine reduces blood pressure by direct vasodilation. Propranolol has indirect effects on vascular tone but (at least initially) reduces pressure by reducing cardiac output. Hydralazine thus evokes reflex sympathetic discharge and increases cardiac force, output, and rate.

Question 99: Relative to Lugol's solution, propylthiouracil has

• propylthiouracil The correct answer is E. Postganglionic sympathetic nerve terminals Explanation: Iodide salts inhibit iodination of tyrosine and thyroid hormone release. The salts also decrease vascularity and size of the hyperplasic thyroid gland. Onset of effect is rapid since both release and synthesis of hormones are inhibited. The effects are transient as the gland "escapes" from the iodide block after several weeks of treatment. Choice D is incorrect; propylthiouracil is the preferred treatment in pregnancy because it is less likely to cross the placenta and into breast milk.

Question 92: In a laboratory study of a new agent, the activity of a membrane-bound enzyme was found to be increased by the drug. Analysis of this enzyme molecule revealed that it is an integral tyrosine kinase, the extracellular domain of which binds ligands while the intracellular domain phosphorylates tyrosine residues. Which of the following receptors communicate their activation by turning on an integral intracellular tyrosine kinase domain?

• protein tyrosine kinase The correct answer is C. Insulin receptors Explanation: Membrane-bound tyrosine kinase receptors are activated by peptides such as insulin and epidermal growth factor, see Table 2-1. | Print Table 2-1 Types of Transmembrane Signaling Receptors. Receptor Type Description Steroid-like Steroids, vitamin D, nitric oxide, and a few other highly membrane-permeable agents cross the membrane and activate intracellular receptors. The effector molecule may be part of the receptor or separate Membrane-spanning receptor-effector enzymes Insulin, epidermal growth factor, and similar agents bind to the extracellular domain of molecules that incorporate tyrosine kinase enzyme activity in their intracellular domains. Most of these receptors dimerize upon activation Membrane receptors that bind intracellular tyrosine kinase enzymes Many cytokines activate receptor molecules that bind intracellular tyrosine kinase enzymes (Janus kinases, JAKs) that activate transcription regulators (signal transducers and activators of transcription, STATs) that migrate to the nucleus to bring about the final effect Ligand-activated or modulated membrane ion channels Certain Na+/K+ channels are activated by drugs: acetylcholine activates nicotinic Na+/K+ channels, serotonin activates 5-HT3 Na+/K+ channels. Benzodiazepines and several other sedative hypnotics allosterically modulate GABA-activated Cl- channels G-protein-coupled receptors (GPCRs) GPCRs consist of 7 transmembrane (7-TM) domains and when activated by extracellular ligands, bind trimeric G proteins and cause the release of activated Gα and Gβγ units. These activated units, in turn, modulate cytoplasmic effectors. The effectors commonly synthesize or release second messengers such as cAMP, IP3, and DAG. GPCRs are the most common type of receptors in the body cAMP, cyclic adenosine monophosphate; IP3, inositol trisphosphate; DAG, diacylglycerol.

Question 96: Which antimalarial drug can cause a dose-dependent toxic state that includes blurred vision, dizziness, flushed and sweaty skin, nausea, diarrhea, and tinnitus?

• quinidine The correct answer is D. Explanation: These dose-related symptoms are characteristic adverse effects of the alkaloids (eg, quinine, quinidine) derived from the bark of the cinchona tree and are termed cinchonism.

Question 74: Ramelteon, a drug prescribed for insomnia, is thought to act in the CNS via

• ramelteon The correct answer is B. Activation of melatonin receptors Explanation: The hypnotic action of ramelteon is thought to be due to its activation of melatonin receptors in the suprachiasmatic nuclei of the CNS.

Question 26: A 16-year-old student has had asthma for 8 yr. The number of episodes of severe bronchospasm has increased recently, and you have been asked to review the therapeutic plan. A long-acting β2-selective agonist that is used as an inhaled therapy for moderate or severe asthma is

• salmeterol The correct answer is C. Explanation: Salmeterol and formoterol are slow-onset, long-acting, selective β2 agonists usually used by inhalation with corticosteroids in asthma prophylaxis. Indacaterol is similar but approved only for COPD.

Question 89: Drugs classified as selective serotonin reuptake inhibitors have minimal clinical efficacy in the treatment of patients who suffer from

• selective serotonin re-uptake inhibitor The correct answer is B. Diminished sexual function and interest Explanation: The selective serotonin reuptake inhibitor (SSRI) class of antidepressants has been shown to have therapeutic value in a wide range of psychiatric dysfunctions ranging from bulimia to obsessive compulsive disorder. One of the adverse effects of the use of SSRIs is diminished sexual function and interest.

Question 18: Ventricular muscle from a cardiac biopsy was prepared for transmembrane potential recording in an isolated muscle chamber. Action potentials were recorded before and after application of drug X. View Full Size | Save Figure | Download Slide (.ppt) Identify drug X from the following list.

• sotalol The correct answer is D. Explanation: The action potential is prolonged without significant slowing of the upstroke, so the drug effect is mainly on potassium channels (group 3 action) and not on both sodium and potassium channels (group 1a action).

Question 1: A 52-year-old woman is admitted to the emergency department with a history of drug treatment for several conditions. Her serum electrolytes are found to be as follows (normal values in parentheses): | Na+ 140 mEq/L (135-145) K+ 6.5 mEq/L (3.5-5.0) Cl− 100 mEq/L (98-107) pH 7.3 (7.31-7.41) This patient has probably been taking

• spironolactone The correct answer is E. Explanation: This patient is hyperkalemic and slightly acidotic. These changes are typical of a K+-sparing diuretic such as spironolactone.

Question 85: Which one of the following is characteristic of succinylcholine?

• succinylcholine The correct answer is D. May cause hyperkalemia Explanation: In phase I block, the action of succinylcholine is not reversed by acetylcholinesterase inhibitors. The drug is metabolized by pseudocholinesterases. It may cause histamine release, and it can cause hyperkalemia.

Question 88: Regarding drugs that relax skeletal muscle which one of the following statements is accurate?

• succinylcholine The correct answer is D. Muscle relaxation caused by succinylcholine is not reversed by acetylcholinesterase inhibitors. Explanation: Baclofen activates GABA receptors, dantrolene blocks the release of calcium from skeletal muscle fibers, halothane enhances skeletal muscle relaxants, and tubocurarine causes histamine release. Acetylcholinesterase inhibitors do not reverse skeletal muscle relaxation caused by succinylcholine.

Question 46: An important difference between nonselective α-receptor antagonists and α1-selective antagonists is that α1-selective antagonists

• tachycardia The correct answer is D Produce less reflex tachycardia Explanation: Reflex tachycardia is a major disadvantage of nonselective α blockers in the treatment of hypertension because the tachycardia is exaggerated by the α2 blockade of nonselective agents. The α1-selective blockers are much less likely to evoke this reflex.

Question 10: A 54-year-old farmer has a 5-yr history of frequent, recurrent, and very painful calcium-containing kidney stones. The patient has hypercalciuria caused by a primary defect in proximal tubule calcium reabsorption. Which of the following is the most appropriate chronic therapy for this man?

• thiazide diuretics The correct answer is E. Explanation: Thiazides increase calcium absorption from the urine into blood, whereas loop diuretics increase calcium excretion from the blood into the urine. None of the other options listed here affects serum calcium. Opioids are indicated for management of severe pain due to kidney stones.

Question 98: A 55-year-old man being treated with a monoamine oxidase inhibitor for resistant major depressive disorder is admitted to the emergency department with a stroke after consuming a large meal in a restaurant. His blood pressure on admission is 180/110 mmHg. When monoamine oxidase inhibitors are used as antidepressant drugs, some patients may suffer hemodynamic effects caused by tyramine in their diet. Which of the following is the primary site of action of tyramine?

• tyramine The correct answer is C. Postganglionic sympathetic nerve terminals Explanation: Tyramine ingested in food is normally metabolized rapidly in the liver by monoamine oxidase. When MAO inhibitors are used in patients, this first-pass effect is blocked and high tyramine concentrations circulate to the tissues. Tyramine acts on noradrenergic nerves (postganglionic sympathetic fibers) to release stores of norepinephrineand may precipitate a hypertensive emergency.

Question 40: Protamine can be used to reverse the anticoagulant effect of which of the following?

• unfractionated heparin The correct answer is D. Explanation: Heparin is negatively charged and will be effectively reversed with the positively charged protamine. There is no antidote for options A through C. If the dose ofwarfarin is too high, vitamin K1 supplements or parenteral phytonadione (vitamin K1) can be added. For urgent reversal of anticoagulation by any drug, fresh frozen plasma may be used.

Question 29: A young woman suffering from myoclonic seizures was receiving effective single-drug therapy. Because she was planning a pregnancy, her physician switched her to an alternative medication with much less potential for teratogenicity. The original antiseizure drug prescribed for this patient was most likely to have been

• valproic acid The correct answer is E. Explanation: Myoclonic seizure syndromes are usually treated with valproic acid. Neural tube defects (spina bifida) are associated with the use of valproic acid during pregnancy.Lamotrigine is approved for adjunctive use but is often used as a sole agent, and several backup drugs are available including topiramate and zonisamide.


Ensembles d'études connexes

U.S History - Chapter 3 Study Guide

View Set

EMT Chapter 20: Allergic Reaction

View Set

PrepU questions for Maternal Health

View Set

learning, memory & consciousness

View Set

Unit 9.3 Persons Subject to State Registration (Series 65)

View Set

Chapter 1: The Nurse's Role in Health Assessment

View Set